Mr. and Mrs. Williams hope to send their daughter to college in twelve years. How much money should they invest now at an interest rate of 8.5% per year,
compounded continuously, in order to be able to contribute $9000 to her education?

Answers

Answer 1

[tex]~~~~~~ \textit{Continuously Compounding Interest Earned Amount} \\\\ A=Pe^{rt}\qquad \begin{cases} A=\textit{accumulated amount}\dotfill & \$9000\\ P=\textit{original amount deposited}\\ r=rate\to 8.5\%\to \frac{8.5}{100}\dotfill &0.085\\ t=years\dotfill &12 \end{cases} \\\\\\ 9000=Pe^{0.085\cdot 12}\implies 9000Pe^{1.02}\implies \cfrac{9000}{e^{1.02}}=P\implies 3245.35\approx P[/tex]


Related Questions

Composing ________________ is another way to compose two functions

Answers

Answer:

is there any options or answers I could see?


Create the equation of a circle that has a center at (1,3) and a radius of 4 units.

Answers

The equation of the circle centered at (1, 3) with a radius of 4 units is:

(x - 1)^2 + (y - 3)^2 = 16

How to write the equation of a circle?

For a circle with the center on the point (a, b) and radius R, the equation is:

(x- a)^2 + (y - b)^2 = R^2

In this case, the center is (1, 3) and the radius is R = 4, so the equation will be, so we just replace these in the general equation that we have above.

(x - 1)^2 + (y - 3)^2 = 4^2

(x - 1)^2 + (y - 3)^2 = 16

If you want to learn more about circles, you can read:

https://brainly.com/question/14283575

please help as soon as possible

Answers

Answer:

D, E

Step-by-step explanation:

In the table, r = 3p, and by converse, p = 1/3r. These options are D and E

Answer:

This is not college level stuff.

But the answer is D. r = 3p and E. p =r/3

Step-by-step explanation:

As you can see in the graph, all the r values are 3 times the p values, 9 = 3 x 3, 21 = 3 x 7, 24 = 3 x 8 and  so on. So, the equation is r = 3p. The same is also vice versa, r/3 = p.

Determine whether AB¯¯¯¯¯¯¯¯
A
B
¯
is a diameter of the circle. Explain your reasoning.

Answers

Answer: D

Step-by-step explanation:

The shared side and sides AC and AD are congruent, as well as the included angles, so the triangles are congruent. This means that by CPCTC, segments BC and BD are congruent, and thus AB bisects CD. Therefore, AB is a diameter.

Solve | x - 6 | = 8

A: x = 14 and x = -14

B: x = 14 and x = -2

C: x = -14 and x = -2

D: x = -14 and x = 2​

Answers

Answer:

B

Step-by-step explanation:

Answer:

x=-2 or x=14

Step-by-step explanation:

since it's absolute value, this actually has two equations:

x-6=8

and

x-6=-8

we solve x-6=8 by adding 6 to both sides which equals x=14

we solve x-6=-8 by also adding 6 to both sides which equals x= -8+6= -2

I hope this helps! and please mark brainliest!

Vicki started jogging. The first time she ran, she ran mile. The second time, she ran mile, and the 516 58 15 16 third time, she ran 16 mile. If she continued this pattern, when was the first time she ran more than 1 mile? Complete the explanation Express the fraction as a mixed number in simplest form. mile(s) every new time she ran. Therefore, the ? time, Vicki had a pastem of adding she ran milles​

Answers

Answer:

Afar fom what i've got from another question on Brainly. I might be wrong but It could Possibly be The 6th Time Vicki Runs!

Step-by-step explanation:

I apologise if this isnt the answer..............

i need help pleasse ​

Answers

X= 30

I don’t know if this is correct or not

Kiara and Mia went to a county fair. They each paid the same admission fee but they were in a different number of rides. Each ride costs the same the total amount they each spent is shown in the table

Answers

let's take admission fee= z

8z+11z=17.50+21.25

19z=38.75

z=38.25/19

z=2.4 (aprox)

Or In case of Kiara 17.50 amount paid in 8 rides

.so 1 ride=17.50/8

=2.1875

same In case of mia

so 1 ride=21.25/11

=1.9318. 

but, they are not match so there is problem in your Questions.

Please help!!!
What is the value of y?
60
45
2V2
Enter your answer, as an exact value in the box
y =

Answers

Answer:

Step-by-step explanation:

Ratio of sides of  45-45-90 triangle =  x : x : x√2

x√2 is the side opposite to angle 90

So, from the picture,

x√2 = 2√2

x = [tex]\frac{2\sqrt{2}}{\sqrt{2}}[/tex]

x = 2

Ratio of sides of 30-60-90 tirangle = a : 2a : a√3

Short side that is oppoiste to 30° = a

Side opposite to 60° = a√3

Hypotenuse (oppoiste to 90°) = 2a

a = 2

y =a√3

  y = 2√3

what is the interquartile rang of the following data set? 9,30,2,48,42,18,81,5,55,73,11

Answers

30 is the real answer

what is the difference of 21 1/4 - 18 2/4​

Answers

Answer:  Difference between 21 1/4 - 18 2/4 =2 3/4

Step-by-step explanation:

You would subtract 21 1/4 - 18 2/4 you would take one from 21 and put it with the 1/4 to make 5/4 and the subtract 5/4 - 2/4=3/4 and 20-18 =2 so 2 and 3/4

Answer my question #4

Answers

Answer:

11. 432,500 12. KF & ND

Step-by-step explanation:

11.

80,000 + 75,000 + 105,000 + 82,500 + 90, 000 = 432,500 people

( it says "about" so it doesn't have to be exact but a reasonable guess)

12. The two stadiums most similar in attendance is Kyle Field and Notre Dame

What is the vertex of the graph of the function f(x) = x2 + 8x − 2 ? (−4, 18) (0, -2) (-8, -2) (−4, −18)

Answers

Answer:

(−4, −18)

Step-by-step explanation:

f(x) = x^2 + 8x - 2

⇒  CONVERT TO VERTEX FORM

≥ f(x) = (x + 4)^2 - 16 - 2

≥ f(x) = (x + 4)^2 - 18

⇒ VERTEX IS AT:

≅ ((−4, −18))

Answer:

The answer is D......

Step-by-step explanation:

How many squares with a side of 3 cm can fit in a 12 cm by 3 cm rectangle

Answers

Answer:

4 squares

Step-by-step explanation:

If the rectangle is only 3 cm wide, only one square will be able to fit in a row. Since the square is 12 cm long, 4 squares can fit. This can be found by 12/3 which = 4

4 sq



Hope this helped.

In which number is the value of the 7 ten times the value of the 7 in the number 17,862

Answers

Answer:

Your answer would require a 7 in the hundreds place

Step-by-step explanation:

This question is worth 100 points if you get this right.

Answers

Answer:

7 nights

Step-by-step explanation:

If he does 1/3 every night then you must turn 2 1/3 into a mixed fraction.

7/3

Then divide.

7/3 by 1/3

your answer is 7/1 or just 7 nights

Just divide

No of nights

2-1/3 ÷1/37/3÷1/37/3×37nights

He need 7 nights

please answer! i need it as its important thank you.

Answers

For the first problem, the solution is as follows:
9y-1/2(4y+20)

1.) Distribute the -1/2 into (4y+20).
9y-2y-10

2.) Combine like terms.
7y-10

3.) Final solution!
7y-10

For the second problem, the answer(s) are solutions A & B.

Solution C cannot be correct because the it does not follow linear algebraic rules (PEMDAS). Solution D is also incorrect because the subtraction sign and division sign change, which does not follow any linear algebraic rules (PEMDAS).

giving brainliest+50 points!!

What is the five-number summary of this data set?

{852, 687, 515, 986, 907, 784, 967, 685, 965}

Drag the numbers into the boxes to correctly complete the five-number summary.

Minimum
Q1
Q2
Q3
Maximum

Answers

Answer:

the 852 is the perfect fit q3

Step-by-step explanation:

The five-number summary of this data set are minimum is 515, maximum value is 986, Q₁ is 686, Q₂ is 852 and Q₃ is 966.

What is Median?

Median of a data set is the element in the middle if the data are arranged in increasing or decreasing order. It is also called second quartile.

Given data set is,

{852, 687, 515, 986, 907, 784, 967, 685, 965}

Arrange this in ascending order first.

{515, 685, 687, 784, 852, 907, 965, 967, 986}

Minimum value = 515

Maximum value = 986

First we have to find the median or the second quartile.

Second quartile, Q₂ = Middle element of the set

Q₂ = 5th element = 852

First quartile = 1/4 (n + 1)th element = 1/4 (9 + 1) = 2.5 th element

Q₁ = (685 + 687) / 2 = 686

Third quartile = 3/4(n + 1)th element = 3/4 (9 + 1) = 7.5th element

Q₃ = (965 + 967) / 2 = 966

Hence the five-number summary of this data set are found.

Learn more about five-number summary here :

https://brainly.com/question/29297089

#SPJ3

Hi can someone may help me with this problem I'm struggling with
ASSP

Answers

Answer:

  see attached

  cost = 33

Step-by-step explanation:

Kruskal's Algorithm is a "greedy" algorithm that adds the next minimum-weight edge to the tree, provided that it does not create a loop.

__

The minimum-weight edge in the graph is DF, with weight 1.

The next minimum-weight edge is GH, with weight 2.

The next minimum-weight edge is AB, with weight 3.

The next minimum-weight edge is EF, with weight 4.

Edge DE is the next lowest-weight, but it creates a loop (DEF), so we ignore it.

BC is the next edge we'll add to our tree.

FG is the next edge we'll add to the tree.

Edge DG creates a loop, so we ignore it.

Edge FH creates a loop, so we ignore it.

CF is the next edge we'll add to the tree. This completes the minimum spanning tree using Kruskal's Algorithm.

Included edges are AB, BC, CF, DF, EF, GF, GH.

The cost of the tree is 3+6+10+1+4+7+2 = 33.

A cylinder has a diameter of 10 inches and a height of 14 inches. what is the volume of the cylinder? use 3.14 for pi?
_______cubic inches

Answers

Answer:

Volume of cylinder is 1099 in³

[tex]\\[/tex]

Step-by-step explanation:

[tex]\\[/tex]

Diameter of cylinder = 10 inchesHeight of cylinder = 14 inches

[tex]\\[/tex]

We know that,

[tex]\dashrightarrow \sf \: \: Radius = \dfrac{Diameter}{2} \\ \\[/tex]

[tex]\dashrightarrow \sf \: \: Radius = \dfrac{10}{2} \\ \\ [/tex]

[tex]\dashrightarrow \sf \: \: { \underline { \boxed{ \pmb{ \sf{ \pink{ Radius = 5 \: inches}}}}}} \\ \\ [/tex]

Using formula:

[tex] \: \: \: \: \: \: {\underline{\boxed{\pmb{\sf{Volume \: of \: cylinder = \pi r^2h}}}}} \: \pink\star \\ \\ [/tex]

Where,

r is radius of the cylinderh is height of the cylinderπ = 3.14

[tex]\\[/tex]

Substituting the required values in the formula:

[tex]\dashrightarrow \sf \: \: Volume = 3.14 × (5)^2 × 14 \\ \\ [/tex]

[tex]\dashrightarrow \sf \: \: Volume = 3.14 \times 25 \times 14 \\ \\ [/tex]

[tex]\dashrightarrow \sf \: \: Volume = 78.5 \times 14 \\ \\ [/tex]

[tex]\dashrightarrow \sf \: \: { \underline{\boxed { \pmb{ \sf{ \pink{Volume = 1099 \: {in}^{3} }}}}}} \\ \\ [/tex]

Hence,

Volume of the cylinder is 1099 in³

find the inverse of f(x)= x-3/2
i dont understand this

Answers

Answer:

the correct answer is shown below

Step-by-step explanation:

hope this helps

In the United States we measure fuel economy in miles per gallon. At the beginning of a trip a car's odometer read 23,450 miles, and the fuel tank was full. At 23,742 miles the fuel tank was refilled with 9.87 gallons of gas. Estimate the number of miles per gallon the car averaged on that portion of the trip.

Answers

Answer:

About 30 miles per gallon of gas

Step by step:

First you subtract 23,742 - 23,450

and then you divide it by 9.87 and then you round

You are calculating the volume of a box which has a width ofx inches, a
length 1 inch less than the width, and a height 2 inches less than the width.
What is the end behavior of the polynomial function that models the volume
as a function of x?
and down
C. down and down
B. down and up
D. up and up
А. up

Answers

Using limits, it is found that the end behavior of the polynomial function for the volume of the rectangular box is:

B. down and up.

What is the volume of a rectangular prism?

The volume of a rectangular prism of length l, width w and heigth h is given by:

V = lwh.

In this problem, we have that the dimensions are given by:

w = x.l = x - 1.h = x - 2.

Hence the volume is given by:

[tex]V = x(x - 1)(x - 2) = x(x^2 - 3x + 2) = x^3 - 3x^2 + 2x[/tex]

What is the end behavior of a function?

It is given by it's limits as x goes to infinity.

Hence:

[tex]\lim_{x \rightarrow -\infty} V(x) = \lim_{x \rightarrow -\infty} x^3 - 3x^2 + 2x = \lim_{x \rightarrow -\infty} x^3 = -\infty[/tex]

[tex]\lim_{x \rightarrow \infty} V(x) = \lim_{x \rightarrow \infty} x^3 - 3x^2 + 2x = \lim_{x \rightarrow \infty} x^3 = \infty[/tex]

Which means that option B is correct.

More can be learned about limits at brainly.com/question/22026723

Write an algebraic expression for the following: 17 more than a number.

Answers

[tex]\qquad \qquad \huge \pink {\sf{☁Answer☁}} \\ \\ [/tex]

Let the number be x

Then,the algebraic expression formed is

[tex] \large \purple{ \boxed { \rm{x + 17}}}[/tex]

It states that you need to write it as 17 more than x This means 17 added on to x .

[tex]\purple{\rule{15mm}{2.9pt}} \red{\rule18mm{2.5pt}} \orange{ \rule18mm{2.5pt}}[/tex]

[tex]\sf{\:мѕнαcкεя\: ♪...}[/tex]

Answer:

x+17

Explanation:

Adding more characters. 'A number is x variable, add 17.. x+17

Trey will attend a -year college, and he is planning his budget.
Answer the following.
(a) Below is an estimate of all his expenses and sources of funding for his -year college education.
For each, select whether it is an expense or a source of funding.

Expense Funding
Scholarships:


Grants:


Housing and food:


Work-study (part-time job):


Personal expenses:


Transportation:


Tuition and fees:


Books and supplies:


(b) What is the total of his estimated expenses?
(c) What is the total of his estimated funding?
(d) Can Trey expect to have enough money for his education? How much is the difference?

Answers

The examples of funding include scholarship, grants, part time job, etc.

What is funding?

Your information is incomplete as the values aren't given. Therefore, an overview will be given.

The funding are needed to cater for school fees and other necessary fees. They include scholarship, grants, and part time job.

The expenses are books, transportation, housing, books, and food.

Learn more about funding on:

https://brainly.com/question/25887038

Divide.
4^2.4^6
O 4-3
O 4^3
O 4-4
O 4^4

The answer is C, 4^4

Answers

Step-by-step explanation:

no, the answer is definitely NOT 4⁴.

4² is smaller than 4⁶, right ?

so, 4²/4⁶ must be smaller than 1.

remember, what we do with exponents when we multiply terms ? we add the exponents.

like 4²×4⁶ = 4⁸

and when we divide terms ? we subtract the exponents.

4²/4⁶ =

[tex] {4}^{ - 4} [/tex]

because 2 - 6 = -4

Find the measures of the interior angles?

Answers

Answer:

Angle C = 35

Step-by-step explanation:

Angles A and B are already given to you, so I will not be explaining how to find them. If you have any questions, let me know.

There are 2 ways to solve this problem; you can either use the sum of the interior angles or isosceles triangles.

Sum of Interior Angles

The interior angles of all triangles equal 180 degrees. We can use this information to create an equation.

First, set up the equation.[tex]110+35+x=180[/tex]Then, combine like terms.[tex]145+x=180[/tex]Next, subtract 145 from both sides[tex]x=35[/tex]

This means that angle C equals 35 degrees.

Isosceles Triangles

The image shows marks on 2 sides. These marks show that the sides are congruent. If a triangle has 2 congruent sides then it is isosceles. Isosceles always have 2 congruent angles as well. In addition, these congruent angles are always the angles opposite of the congruent sides.

In this case, the angles opposite of the congruent sides are angle A and angle C. Thus, angles A and C are congruent. If angle A is equal to 35, then angle C must also be 35.

I did half the answer but what is e= ?​

Answers

Answer:

See below.

Step-by-step explanation:

e=12c+6

e=12(1)+6

e=12+6

e=18

e=12(2)+6

e=24+6

e=30

e=12(3)+6

e=36+6

e=42

e=12(4)+6

e=48+6

e=54

-hope it helps

Find the value of x. AB=20 BC=6 CD=8​

Answers

Step-by-step explanation:

there is the rule

outside length × (outside length + inside length) is the same for all lines originating from the same point.

so, we have

CD(CD + x) = BC(BC + AB) = 6(6+20) = 6×26 = 156

8(8 + x) = 156

8 + x = 19.5

x = 11.5

what should be the rate of interest to earn an 45000 as interest wale rupees 50000 easy in investment for 6 year​

Answers

Answer:

5

Step-by-step explanation:

Other Questions
In chemiosmosis in mitochondria, protons flow from the intermembrane space into the matrix, whereas in chemiosmosis in chloroplasts, protons flow from:. Please Help! What is the measure of the reference angle that corresponds to an angle of measure 278 GUYS I NEED HELP ASAP IVE BEEN STUCK ON THIS MATH PROBLEM The cylinder shown has a volume of 695 cubic inches.What is the radius of the cylinder? NO LINKS!!Part 2: Figure A is a dilated image of Figure B. Find the scale factor. #3 and 4 A store sells two cone-shaped funnels. What is the height of each funnel? Round your answer to the nearest tenth(1 pt=28.875 in)45 in5 inVolume 0.5 pintVolume 1 pintThe height of the smaller funnel is aboutinches.The height of the larger funnel is aboutinches. The area of a circle is 100 ft. What is the circumference, in feet? express your answer in terms of \pi Y=x-12, y=14 what is x I need someone to answer fast please Byron has a bad track record of playing with fire in the house and Momma strongly disapproves because she believes Byrons games put the whole family in danger. Discuss whether Mommas reaction when she discovers Byron burning tissue in the bathroom is extreme. Is Mommas decision to burn Byron as punishment a just choice? Why? Was it appropriate for Joetta to intervene to save Byron? Why?Watsons go to Birmingham From time to time, the Supreme Court has extended civil rights protections to groups that historically have been deniedrights guaranteed to other Americans. Sometimes the Supreme Court has done this by interpreting and applyingprinciples found in existing Constitutional amendments.Which legal principle did the Supreme Court use to extend the Bill of Rights guarantees to state laws and to broaden civilrights protections for minorities?A the "two-thirds clause" of the 12th AmendmentB the "punishment clause" of the 13th Amendmentthe "due process clause" of the 14th Amendmentthe "apportionment clause" of the 16th Amendment E-mail is an temporary message medium. please select the best answer from the choices provided t f Select the correct answers from the drop-down menus. Read the excerpt. Then choose the correct way to complete each sentence. Wilson sees as the key to achieving peace. A quotation that supports this conclusion is Answer the question based on the graph What is the Y-intercept?What is the asymptote? Eight striped ties and 6 polka dot ties are on arack in a dark closet. What is the probabilityof drawing out one polka dot tie? Write youranswer as a decimal rounded to two decimalplaces. PLEASE HELPWhich is the graph of f(x) = 1/4 (4)x?No links Help me and answer correctly and if you do then I will mark you brainliest!!! Mia gets 15 out of 18 questions correct on a test. What is the ratio of correctanswers to the total number on the test in simplest form? A triangle has two sides of length 19 and 18. What is the largest possible whole-number length for the third side? We have to finish dividing the cytoplasm and then back to work. Cant wait to get back to normal Pete used sediments that were all the same shape and size in this demonstration. How would varying the size of the sediments affect the porosity (assuming the total volume of sediment remains the same)? Write in complete sentences using good grammar. Please help I need to complete this today